Limits involving exponential functions

Click For Summary
The discussion focuses on understanding the application of L'Hôpital's rule to limits involving exponential functions, particularly the form 0^0. It clarifies that the limit of e^f(x) can be taken inside the exponential due to the continuity of the exponential function. However, this is contingent on the behavior of f(x) at the point of interest, as f(x) may not be continuous at that point. Participants express a desire for a formal proof demonstrating this property, similar to established proofs for basic limit properties. Overall, the conversation emphasizes the importance of continuity in evaluating limits involving exponential functions.
ahmadmz
Messages
62
Reaction score
0

Homework Statement



I was studying L'Hopital's rule and how to deal with indeterminate forms of the type 0^0.

It's not clear to me how lim e^f(x) = e^lim f(x).

In wikipedia http://en.wikipedia.org/wiki/L%27H%C3%B4pital%27s_rule" (under other indeterminate forms)
it says "It is valid to move the limit inside the exponential function because the exponential function is continuous".

But that would mean lim(x->a) e^f(x) = e^f(a).

Homework Equations





The Attempt at a Solution


 
Last edited by a moderator:
Physics news on Phys.org
\lim_{x \rightarrow a}e^{f(x)} = e^{\lim_{x \rightarrow a} f(x)}


But for limx->af(x)=f(a) would only occur if f(x) is continuous at x=a
 
Last edited:
Remember that f(x) may have issues at a, as mentioned above, even when it is part of e^f(x). You can move the limit inside the exponential, because the exponential itself doesn't have problem spots ("is continuous everywhere"), so it is only the f(x) inside that you have to deal with regarding the limit.
 
Yea it makes sense, but is there some way to show this as a proof?
For instance the basic limit properties such as lim(f + g) = lim f + lim g are proved.
 
It should be straightforward to show from the limit definition of "continuous function" that if g is continuous at L and \lim_{x\rightarrow a} f(x) = L, then \lim_{x\rightarrow a} g(f(x)) = g(L).
 
Question: A clock's minute hand has length 4 and its hour hand has length 3. What is the distance between the tips at the moment when it is increasing most rapidly?(Putnam Exam Question) Answer: Making assumption that both the hands moves at constant angular velocities, the answer is ## \sqrt{7} .## But don't you think this assumption is somewhat doubtful and wrong?

Similar threads

  • · Replies 5 ·
Replies
5
Views
2K
Replies
8
Views
2K
  • · Replies 19 ·
Replies
19
Views
2K
  • · Replies 3 ·
Replies
3
Views
2K
  • · Replies 5 ·
Replies
5
Views
1K
  • · Replies 5 ·
Replies
5
Views
2K
  • · Replies 9 ·
Replies
9
Views
2K
  • · Replies 7 ·
Replies
7
Views
1K
  • · Replies 8 ·
Replies
8
Views
2K
  • · Replies 3 ·
Replies
3
Views
2K